Bạn chưa đăng nhập. Vui lòng đăng nhập để hỏi bài

Những câu hỏi liên quan
Ngô Thị Lan Anh
Xem chi tiết
Wan
Xem chi tiết
Ng Thu Trà
Xem chi tiết
๖ۣۜDũ๖ۣۜN๖ۣۜG
24 tháng 6 2021 lúc 17:31

Do VP là số lẻ

<=> 2x + 5y + 1 là số lẻ và \(2^{\left|x\right|}+y+x^2+x\) là số lẻ

<=> y chẵn và \(2^{\left|x\right|}+y+x\left(x+1\right)\) là số lẻ 

=> \(2^{\left|x\right|}\) là số lẻ (do y chẵn và x(x+1) chẵn)

=> x = 0

PT <=> \(\left(5y+1\right)\left(1+y\right)=105\)

<=> y = 4 (thử lại -> thỏa mãn)

KL: x = 0; y = 4

Thuhuyen Le
Xem chi tiết
Nguyen Minh
Xem chi tiết
Vuy năm bờ xuy
3 tháng 6 2021 lúc 2:05

\(\sqrt{x+y+3}+1=\sqrt{x}+\sqrt{y}\)

Bình phương 2 vế, ta có:

\(x+y+3+1=x+y\)

\(x+y+3+1-x-y=0\)

\(4=0\) (vô lý)

Vậy phương trình vô nghiệm

-Chúc bạn học tốt-

Đặng Khánh
3 tháng 6 2021 lúc 8:49

(x,y) hoán vị của (4,9) . có vẻ hoạt động

Rhider
Xem chi tiết
Nguyễn Việt Lâm
24 tháng 1 2022 lúc 7:48

- Với \(x=1\Rightarrow y=1\)

- Với \(x>1\Rightarrow y>1\)

\(\Rightarrow3^x=2^y+1\)

Do \(y>1\Rightarrow2^y⋮4\Rightarrow2^y+1\equiv1\left(mod4\right)\) \(\Rightarrow3^x\equiv1\left(mod4\right)\)

Nếu \(x=2k+1\Rightarrow3^x=3^{2k+1}=3.9^k\equiv3\left(mod4\right)\) (ktm) 

\(\Rightarrow x=2k\Rightarrow3^{2k}-1=2^y\)

\(\Rightarrow\left(3^k-1\right)\left(3^k+1\right)=2^y\)

\(\Rightarrow\left\{{}\begin{matrix}3^k-1=2^a\\3^k+1=2^b\end{matrix}\right.\) với \(b>a\Rightarrow2^b-2^a=2\)

\(\Rightarrow2^a\cdot\left(2^{b-a}-1\right)=2\Rightarrow2^a=2\Rightarrow\left\{{}\begin{matrix}a=1\\b=2\end{matrix}\right.\)

\(\Rightarrow3^k-1=2\Rightarrow k=1\Rightarrow x=2\Rightarrow y=3\)

Vậy \(\left(x;y\right)=\left(1;1\right);\left(2;3\right)\)

king
Xem chi tiết
Như Dương
Xem chi tiết
Như Dương
29 tháng 8 2021 lúc 10:15

ai giúp em bài1 và phần b bài 2 với ạ

 

nguyen minh quan
Xem chi tiết